Avec deux orthocentres

Bonjour à tous,
j'ai découvert pappus lors de la lecture d'un de ses articles dans la revue Tangente au sujet de....
Aussi, je lui dédie ce problème...

1. ABC un triangle acutangle
2. P un point de [BC]
3. Q l'isotome de P relativement à [BC]
4. Pp, Pq les perpendiculaires à (BC) issues resp. de P, Q
5. E, F les points d'intersection resp. de Pp et (AC), Pq et (AB)
6. M le point d'intersection de (PF) et (QE)
7. H1, H2 les orthocentres resp. des triangles BFP, CEQ.

Question : (AM) est perpendiculaire à (H1H2).

Joyeux Noël et Meilleurs Voeux pour 2021
Sincèrement
Jean-Louis114352

Réponses

  • Bonjour,

    Avec Morley circonscrit:
    % Jean-Louis Ayme - 13/12/2020 - Avec deux orthocentres 
    
    clc, clear all, close all
    
    syms a b c
    syms aB bB cB % Conjugués
    
    aB=1/a;
    bB=1/b;
    cB=1/c;
    
    %-----------------------------------------------------------------------
    
    syms t real
    
    p=b+t*(c-b);  % Un point P de (BC)
    pB=bB+t*(cB-bB);
    
    q=b+c-p; % L'isotomique Q de P
    qB=bB+cB-pB;
    
    [ppbc qpbc rpbc]=DroitePerpendiculaire(p,b,c,pB,bB,cB); % Droite (Pp)
    [pqbc qqbc rqbc]=DroitePerpendiculaire(q,b,c,qB,bB,cB); % Droite (Pq)
    
    [e eB]=IntersectionDeuxDroites(ppbc,qpbc,rpbc,1,a*c,-a-c);
    [f fB]=IntersectionDeuxDroites(pqbc,qqbc,rqbc,1,a*b,-a-b);
    
    % On trouve:
    
    e = (2*a*b - a*c + b*c - 2*a*b*t + 2*a*c*t)/(a + b);
    f = (2*a*c - a*b + b*c + 2*a*b*t - 2*a*c*t)/(a + c);
    
    [ppf qpf rpf]=DroiteDeuxPoints(p,f,pB,fB);
    [pqe qqe rqe]=DroiteDeuxPoints(q,e,qB,eB);
    
    [m mB]=IntersectionDeuxDroites(ppf,qpf,rpf,pqe,qqe,rqe);
    
    % On trouve:
    
    m = (2*a^2 - 3*a^2*t + a*b*t + a*c*t + b*c*t)/(2*a);
    
    [h1 h1B]=Orthocentre(b,f,p,bB,fB,pB);
    [h2 h2B]=Orthocentre(c,e,q,cB,eB,qB);
    
    h1=Factor(h1);
    h2=Factor(h2);
    
    % On trouve:
    
    h1 = -(a*b - 2*a*c + b*c + c^2*t - 3*a*b*t + 3*a*c*t - b*c*t)/(a - c);
    h2 = -(a*c - 2*a*b + b*c + b^2*t + 3*a*b*t - 3*a*c*t - b*c*t)/(a - b);
     
    h1h2=Factor(h2-h1);
    h1h2B=Factor(h2B-h1B);
    
    am=Factor(m-a);
    amB=Factor(mB-aB);
    
    Nul=Factor(am*h1h2B+amB*h1h2) %Égal à 0, donc c'est gagné
    
    Cordialement,

    Rescassol
  • Bonjour,

    Cordialement,

    La droite $(AM)$ est indépendante du choix de $P$ sur $(BC)$.
    Elle coupe $(BC)$ en $A_1\left(\dfrac{s_2-a^2}{2a}\right)$, point de contact de la deltoïde de Steiner avec $(BC)$. Ce point et les deux autres obtenus par permutation circulaire forment le triangle podaire $A_1B_1C_1$ du point de De Longchamps $L(-s_1)$ du triangle $ABC$.
    Cette droite et les deux droites obtenues par permutation circulaire sont concourantes en l'isotomique $X_{69}(x_{69})$ de l'orthocentre $H(s_1)$ du triangle $ABC$ avec $x_{69}=\dfrac{s_1^2s_2+3s_1s_3-4s_2^2}{s_1s_2-9s_3}$.

    Cordialement,

    Rescassol
  • Mon cher Pappus,
    aucune initiative...as-tu une petite idée?

    Sincèrement
    Jean-Louis
  • Mon cher Jean_Louis
    Tout d'abord je te souhaite un joyeux Noël et une Bonne Année et je te remercie de tes bons voeux.
    Bien sûr j'ai commencé à regarder ta nouvelle énigme mais je suis resté sec pour le moment
    Ce que raconte Rescassol, que je salue et à qui j'adresse tous mes voeux, me semble fort intéressant. Il y a sans doute quelque chose à faire avec les droites de Simson.
    Amicalement
    [small]p[/small]appus
  • Mon cher Pappus, bonjour à tous,

    pour aller de l'avant...
    considérons les symétriques U, T resp. de H1, H2 par rapport au milieu M de [BC]...

    Sincèrement
    Jean-Louis
  • Bonjour à tous, joyeux Noël et Meilleurs Voeux pour 2021,

    J'utilise les coordonnées barycentriques.

    Le triangle de référence $ABC$ :

    $A,B,C\simeq \left[\begin{array}{c} 1\\ 0\\ 0\end{array}\right],\left[\begin{array}{c} 0\\ 1\\ 0\end{array}\right],\left[\begin{array}{c} 0\\ 0\\ 1\end{array}\right]$

    $P$ un point de [BC] :

    $P\simeq \left[\begin{array}{c}0\\ 1-t\\t \end{array}\right]$

    L'isotome Q de P relativement à [BC] :

    $Q\simeq \left[\begin{array}{c}0\\ t\\1-t \end{array}\right]$

    Pp, Pq les perpendiculaires à (BC) issues resp. de P, Q :

    $P_p : (-b^2 + c^2 + a^2 (1 - 2 t))x -2 a^2 ty -2 a^2 (-1 + t)z=0$

    $P_q : (-b^2 + c^2 + a^2 (-1 + 2 t))x+ 2 a^2 (-1 + t)y+ 2 a^2 tz=0.$

    E, F les points d'intersection resp. de Pp et (AC), Pq et (AB) :

    $E \simeq \left[\begin{array}{c} 2 a^2 (-1 + t)\\0\\-b^2 + c^2 + a^2 (1 - 2 t)\end{array}\right]$

    $F \simeq \left[\begin{array}{c}2 a^2 (-1 + t)\\ b^2 - c^2 + a^2 (1 - 2 t)\\ 0\end{array}\right]$

    M le point d'intersection de (PF) et (QE) :

    $M\simeq \left[\begin{array}{c} -2 a^2 (-1 + t)\\(a^2 - b^2 + c^2) t\\ (a^2 + b^2 - c^2) t\end{array}\right]$

    H1, H2 les orthocentres respectifs des triangles BFP, CEQ :

    $H_1, H_2\simeq \left[\begin{array}{c} 2 a^2 (a^2 - b^2 + c^2) (-1 + 2 t)\\ a^4 (1 - 3 t) + (b^2 - c^2)^2 (-1 + t) +
    2 a^2 (b^2 + c^2) t\\-a^4 t - (b^2 - c^2)^2 t + a^2 (c^2 (4 - 6 t) + 2 b^2 t)\end{array}\right],\left[\begin{array}{c} -2 a^2 (a^2 + b^2 - c^2) (-1 + 2 t)\\ a^4 t + (b^2 - c^2)^2 t + a^2 (-2 c^2 t + b^2 (-4 + 6 t))\\ -(b^2 - c^2)^2 (-1 + t) - 2 a^2 (b^2 + c^2) t + a^4 (-1 + 3 t)\end{array}\right]$

    Question : (AM) est-elle perpendiculaire à (H1H2) ?

    $(AM) \simeq \left[\begin{array}{c} 0\\ -a^2 - b^2 + c^2\\ a^2 - b^2 + c^2\end{array}\right]=U$

    $(H_1H_2) \simeq \left[\begin{array}{c} (b^2 - c^2)^2 + 2 a^2 (b^2 + c^2) + a^4 (1 - 4 t)\\ 2 a^2 (b^2 - c^2 + a^2 (1 - 2 t))\\ 2 a^2 (-b^2 + c^2 + a^2 (1 - 2 t))\end{array}\right]=V$

    On a

    $U^{t} \times \left[\begin{array}{c} 2a^2 & - a^2 - b^2 + c^2 & - a^2 + b^2 - c^2\\ - a^2 - b^2 + c^2 & 2b^2 & a^2 - b^2 - c^2\\ - a^2 + b^2 - c^2 & a^2 - b^2 - c^2 & 2c^2\end{array}\right] \times V=0$

    Ainsi $(AM)$ est perpendiculaire à $(H_1H_2).$

    Amicalement
  • Bonjour Jean-Louis

    Quelques éléments incomplets :
    $H_1$ est l'orthocentre de BFP, donc F est l'orthocentre de $BH_1P$
    même chose pour $H_2$

    On retrouve en-dessous de BC la même figure qu'au-dessus à une symétrie près autour du milieu O de BC et une affinité par rapport à BC.

    En permutant P et Q, on passe de E et F à E' et F', et de M à M'. Le théorème de Désargues appliqué à E'FPF'EQ montre que A, M et M' sont alignés. Il reste à montrer que le milieu de MM' est fixe et que c'est le milieu de AG' (la droite AG' est la conjuguée isotomique de la hauteur AG) : cela résulte de la "pseudo-symétrie" indiquée ci-dessus. Soit $N=H_1P \cap H_2Q$ : sa projection orthogonale sur BC est confondue avec celle de M'.

    Il reste aussi à montrer que $H_1H_2$ reste parallèle à une direction fixe. On se place alors dans le cas particulier où P est en B : A' est sur le cercle circonscrit à ABC et on prouve facilement l'orthogonalité de $H_1H_2$ et de AG' par les angles.

    Joyeux Noël
    PL114680
  • Bonjour,

    • Notons X le milieu de [BC]
    et U, T les symétriques de H1, H2 par rapport à M.

    • Conclusion partielle : (H1H2) // (UT)

    • Les triangles BFP et CEQ sont * orthologiques, U et T en sont les centres

    * perspectifs, d'axe (AM).

    Nous pouvons conclure...

    Sincèrement
    Jean-Louis
  • Mon cher Pappus,

    après ma dernière intervention, je te laisse le plaisir de citer le théorème que tu connais si bien...
    J'attends...

    Sincèrement
    Jean-Louis
  • Bonjour,

    https://jl.ayme.pagesperso-orange.fr/ vol. 49 Orthique 15, p. 26...

    Sincèrement
    Jean-Louis
Connectez-vous ou Inscrivez-vous pour répondre.